Zastanawiająca implikacja

Definicja klasyczna. Prawdopodobieństwo warunkowe i całkowite. Zmienne losowe i ich parametry. Niezależność. Prawa wielkich liczb oraz centralne twierdzenia graniczne i ich zastosowania.
mwrooo
Użytkownik
Użytkownik
Posty: 215
Rejestracja: 18 cze 2013, o 21:58
Płeć: Mężczyzna
Lokalizacja: Kuczbork
Podziękował: 34 razy

Zastanawiająca implikacja

Post autor: mwrooo »

Witam, czy ta implikacja jest prawdziwa? Jeśli tak, to proszę o wskazówkę:
\(\displaystyle{ \int_{Y<0}^{}Y\;dP \ge 0 \Rightarrow Y \ge 0}\).
tometomek91
Użytkownik
Użytkownik
Posty: 2959
Rejestracja: 8 sie 2009, o 23:05
Płeć: Mężczyzna
Lokalizacja: Wrocław
Podziękował: 281 razy
Pomógł: 498 razy

Zastanawiająca implikacja

Post autor: tometomek91 »

Nie. Rzuc moneta i daj 1 gdy orzel i -1 gdy reszka i sprawdz
mwrooo
Użytkownik
Użytkownik
Posty: 215
Rejestracja: 18 cze 2013, o 21:58
Płeć: Mężczyzna
Lokalizacja: Kuczbork
Podziękował: 34 razy

Zastanawiająca implikacja

Post autor: mwrooo »

Uważam że jest to zły przykład i w sumie chyba juz doszedłem dlaczego faktycznie tak jest. Ta implikacja jest prawdziwa.
a4karo
Użytkownik
Użytkownik
Posty: 22210
Rejestracja: 15 maja 2011, o 20:55
Płeć: Mężczyzna
Lokalizacja: Bydgoszcz
Podziękował: 38 razy
Pomógł: 3755 razy

Zastanawiająca implikacja

Post autor: a4karo »

Zgadza się. (Oczywiśćie \(\displaystyle{ \geq}\) oznacza \(\displaystyle{ \geq}\) prawie wszędzie.)
mwrooo
Użytkownik
Użytkownik
Posty: 215
Rejestracja: 18 cze 2013, o 21:58
Płeć: Mężczyzna
Lokalizacja: Kuczbork
Podziękował: 34 razy

Zastanawiająca implikacja

Post autor: mwrooo »

\(\displaystyle{ \int_{Y<0}^{}Y\;dP > 0 \Rightarrow Y > 0}\).
Ale taka implikacja już nie jest prawdziwa prawda?
a4karo
Użytkownik
Użytkownik
Posty: 22210
Rejestracja: 15 maja 2011, o 20:55
Płeć: Mężczyzna
Lokalizacja: Bydgoszcz
Podziękował: 38 razy
Pomógł: 3755 razy

Zastanawiająca implikacja

Post autor: a4karo »

Jest prawdziwa, bo poprzednik jest zawsze fałszywy (całka z ujemnej funkcji nie może być dodatnia).
mwrooo
Użytkownik
Użytkownik
Posty: 215
Rejestracja: 18 cze 2013, o 21:58
Płeć: Mężczyzna
Lokalizacja: Kuczbork
Podziękował: 34 razy

Zastanawiająca implikacja

Post autor: mwrooo »

Tak, zgadza się, chodziło mi o to, że poprzednik jest fałszywy, dzięki!
ODPOWIEDZ